Quantcast
  • Register
PhysicsOverflow is a next-generation academic platform for physicists and astronomers, including a community peer review system and a postgraduate-level discussion forum analogous to MathOverflow.

Welcome to PhysicsOverflow! PhysicsOverflow is an open platform for community peer review and graduate-level Physics discussion.

Please help promote PhysicsOverflow ads elsewhere if you like it.

News

PO is now at the Physics Department of Bielefeld University!

New printer friendly PO pages!

Migration to Bielefeld University was successful!

Please vote for this year's PhysicsOverflow ads!

Please do help out in categorising submissions. Submit a paper to PhysicsOverflow!

... see more

Tools for paper authors

Submit paper
Claim Paper Authorship

Tools for SE users

Search User
Reclaim SE Account
Request Account Merger
Nativise imported posts
Claim post (deleted users)
Import SE post

Users whose questions have been imported from Physics Stack Exchange, Theoretical Physics Stack Exchange, or any other Stack Exchange site are kindly requested to reclaim their account and not to register as a new user.

Public \(\beta\) tools

Report a bug with a feature
Request a new functionality
404 page design
Send feedback

Attributions

(propose a free ad)

Site Statistics

205 submissions , 163 unreviewed
5,047 questions , 2,200 unanswered
5,345 answers , 22,709 comments
1,470 users with positive rep
816 active unimported users
More ...

  No closed loop in a fractal laser

+ 0 like - 0 dislike
1080 views

You can make a medium lase, if it contains fractal metallic fragments, and the lasing depends on, a photon always making a path back to its original starting point, or making closed loops.  What would happen in such a medium if you could somehow prevent closed loops from ever occurring?

asked Jul 25, 2018 in Experimental Physics by anonymous [ no revision ]

1 Answer

+ 0 like - 0 dislike

I found this in the first comment: "Note that only electrostatic field lines cannot form closed loops. If the magnetic flux is changing, it will induce a current via closed loops of electric field. – jvriesem"    at https://physics.stackexchange.com/questions/105592/why-cant-electrostatic-field-lines-form-closed-loops

The motivation for it is that I noticed in a square grid like graph paper, and on a cellular automata with a square grid (an explosive rule in a normal automata like Life, but explosive so that the cells flicker on and off across the entire screen and fill the screen), is that if you measure a cell with a ruler, the diagonal length is root 2 times longer than the edge, but the counting of the diagonal cells in a group of 9 in a square grouping, is the same as the two legs (or the counting length of cells in a 45-45-90 "triangle" is the same as the legs).  However, when zoomed out in golly, if the eye sees something moving, diagonal elements travel faster than vertical or horizontal motion, due to a constant update with no preference in terms of the speed that a cell turns on or off, so that a diagonal moving group of cells must travel faster because they have to travel a longer distance in terms of real distance.  I thought that by having a variable increase in how fast a cell turns on, ie it turns on faster if a cell is born to the right of a cell.  Then (as it stands with no variable it causes small rotations in small collective cells), with this way, since cells are born faster and propagate faster towards the right horizontal direction, there is now a bias in a vector to the right and a positive or negative vector along either diagonal.  If you draw a vector sum of a vector to the right and a diagonal vector, before the diagonal vector was longer since you before didn't have bias in the right direction as well, so that now, the "bias" is now only in the up-down direction.  So there should be a separation of elements in the up-down direction as opposed to the diagonal, so instead of circular rotations, there might be planar twisting of rotations.  It may be both up-down and left right ie a cell directly above (not one above and to the left or right a diagonal cell) cells turn on faster.  Also, to generate the correct diagonal (1.4 ... an irrational number) the faster update (the range of cells), may need to be continuous or linked to an analog crystal in an arduino chip like this: https://www.fpga4fun.com/oscillators.html connected to the computer, to generate the correct irrational update (as well as handling the range of updates, and how to make the update rules for the automata).

answered Jul 25, 2018 by anonymous [ revision history ]
edited Jul 25, 2018

Your answer

Please use answers only to (at least partly) answer questions. To comment, discuss, or ask for clarification, leave a comment instead.
To mask links under text, please type your text, highlight it, and click the "link" button. You can then enter your link URL.
Please consult the FAQ for as to how to format your post.
This is the answer box; if you want to write a comment instead, please use the 'add comment' button.
Live preview (may slow down editor)   Preview
Your name to display (optional):
Privacy: Your email address will only be used for sending these notifications.
Anti-spam verification:
If you are a human please identify the position of the character covered by the symbol $\varnothing$ in the following word:
p$\hbar$ysicsOverfl$\varnothing$w
Then drag the red bullet below over the corresponding character of our banner. When you drop it there, the bullet changes to green (on slow internet connections after a few seconds).
Please complete the anti-spam verification




user contributions licensed under cc by-sa 3.0 with attribution required

Your rights
...